First sentence I diagrammed as: wind & below 84-> pleasant. The second part of this question I diagrammed as : above 84 or no wind ->oppressive. I incorrectly got answer choice (B) it has something to do the the humidity but I was able to add ...
http://7sage.com/lsat_explanations/lsat-28-section-3-question-12/
I really do not understand why the answer is C, and the question stem also seems to be very confusing. Are we suppose to find one option that is "must be false", or there are four " ...
https://7sage.com/lsat_explanations/lsat-41-section-3-question-23
I understand the reasoning in the video, that just because X is preceded by Y, it doesn’t mean that Y is the necessary condition.
https://7sage.com/lsat_explanations/lsat-67-section-3-game-4/
I was not sure about the question sentence...it says "If three subzones in all are designated for retail use and a subzone in Z2 is designated for housing, then which one of the following ...
I cannot for the life of me figure out why C and D are wrong, can give a coherent detailed explanation for why they're wrong?
https://7sage.com/lsat_explanations/lsat-40-section-3-question-23/
hello everyone, this is a question that was asked but there was no clear answer. The answer choice for A, if you take the contraceptive it matches the flaw. Is A only ...
Hi everyone! For this question, I understand how answer choice B wrecks the argument, making it a solid necessary assumption. It was my original answer choice, but I thought I was being tricked at the back end of the test because it seemed like a ...
My big issue with this question is about why B is the correct answer. It seems to equate "exploiting" with "destroy" and I'm not sure how reasonable of an assumption that is to make. Since this is a logically inferred question, I assumed that the right ...
would it be a good way to think alternative to JY's explanation that answer choice B is wrong merely in having few as the existential indicator rather than a universal ...
Ok, so I got this MBT question wrong. I initially was going to go with AC D (the right choice) but was turned off from the second part of the answer. The whole thing reads:
"More money is spent on microwave food products that take three ...
I am confused as to why answer c is a better answer choice than a. They seem very similar, but is the answer c because there are a range of numbers opposed to using the same number (2) in answer choice a, or is there something bigger that I am missing?
Hi everyone, question on how to approach this question. The question gives two arguments, Jane's and Maurice's. The question stem states, "Which one of the following, if true, most strengthens Jane's argument?" So after reading this question stem we know ...
Correct answers in MSS questions are almost always one of two types: restatement of an idea from the stimulus or the conclusion of the stimulus (which was left unsaid in the stimulus).
In this necessary assumption question, gardeners who plant according to the phases of the moon despite the fact that the phases of the moon do not affect how well plants grow. We are then told that gardeners who plant during the first warm spell of the ...
I find this strengthening question particularly tricky. Can anyone help explain why E) is the correct answer? Also explaining why C) isn't would be helpful!
So the correct answer here was E. I can see why all the wrong answers are wrong, but I had a really hard time accepting that E is right because I saw "doctors" here as referring to ALL doctors (and I don't think we know anything about ...